PT49.S2.Q11 - cholesterol, which is a known factor in coronary heart disease

Luluc1234Luluc1234 Alum Member
edited January 2016 in Logical Reasoning 150 karma
http://7sage.com/lsat_explanations/lsat-49-section-2-question-11/
While the right answer choice is very clear to me, it would be great if someone coule help me to clarify the reasoning structure for both the stimulus and answer choice (B)?

My Breakdown:

Stimulus:
Aerobic exercise & Women---correlates--- higher HDLs
Aerobic exercise & Women---correlates--- lower risk of Heart Disease& Stroke
--------------------
higher HDLs --cause--> lower risk of Heart Disease& Stroke

This reasoning seems to be making the correlation-causation error. However, can we rightfully assume since A correlates with both B and C , that B correlates with C?

*My intuition would say NO, because A most B, A most C, only implies B some C.

Similarly,

1. In the answer choice (B), the strengthening seems to rely on the same logic that Lower level of HDLs (absence of cause) correlates with higher risk of Heart Disease& Stroke (absence of effect) because both correlates with overweight.

2. some suggests the idea of overweight as the strengthener for Aerobic exercise.They reason that correlation between AE and lower risk HD&S is strengthened because Overweight (absence of Aerobic exercise) correlates with higher risk of HD&S (absence of lower risk HD&S). But first can we assume that overweight generally implies absence of aerobic exercise? and second if the emphasis is on overweight, this answer choice would seem like a premise booster.

Comments

  • nicole.hopkinsnicole.hopkins Inactive Sage Inactive ⭐
    7965 karma
    @Luluc1234 said:
    However, can we rightfully assume since A correlates with both B and C , that B correlates with C?
    Watch the video for PT50S4Q19 (my nemesis question for PT50) for discussion of this kind of positive correlation argument. http://7sage.com/lsat_explanations/lsat-50-section-4-question-19/

    I'm not really sure how to deal with the question of existential quantifiers in terms of correlation as you're asking it ... Tap in @"Jonathan Wang" (or shine his LSAT/bat signal).

    Aside from that ...

    The first thing I would say is that the conclusion says "tentatively conclude": the author already recognizes that the argument is more or less one of correlations lining up, possibly indicating a common cause. It's like braiding together several cords which, weak on their own, might support a potentially less tentative conclusion. We want to help the author make his argument by:

    1) Saying C/B does not cause A
    2) A does cause C and B (something
    3) A third cause does not account for A, B, and C
    4) That there is indeed a relationship (as indicated by additional evidence)

    If this were a weakening question, we would want to say:

    1) C/B causes A
    2) A does not cause C and/or B because of competing/conflicting evidence
    3) A third factor causes A/B/C
    4) There's no relationship

    So since this is an EXCEPT question, we want to find something that either fails to do 1-4 of the first list, or actually does 1-4 of the last list (or is just out of scope).

    I would say that B is basically doing #4 from the first list: adding yet another correlation to demonstrate that there is a relationship between heart disease/stroke and lack of HDL's. Adding another cord to the still tentative and imperfect relationship—but one cord more to keep it from snapping in supporting the weight of the conclusion. It's not a premise booster—it's adding an additional (closely related, indicating and additional piece of evidence) premise.

    C is out of scope and shows a misunderstanding of the relationship being discussed. (Cue JY voice in my head) So what if they're less easily removed? Has nothing to do with the argument.
Sign In or Register to comment.